Search found 12 matches


parallel and pronoun issue

A majority of the international journalists surveyed view nuclear power stations as unsafe at present but that they will, or could, be made sufficiently safe in the future. (A) that they will, or could, (B) that they would, or could, (C) they will be or could (D) think that they will be or could (E)...

by gmatwarroom

Sun Feb 17, 2013 2:35 pm
Forum: Sentence Correction
Topic: parallel and pronoun issue
Replies: 4
Views: 1850

The statement is only sufficient if it along with the given information can conclusively answer the question. I think, I would rephrase it to make it more understandable -- The statement is only sufficient if it along with the given information can get UNIQUE value/answer to the question. Also, I t...

by gmatwarroom

Sat Sep 29, 2012 1:25 pm
Forum: Data Sufficiency
Topic: confused with this DS
Replies: 6
Views: 1518

thanks everyone for solving the equations. However, my issue was not really "solving" each of the equations but solving the problem i.e. deriving the conclusion. As I mentioned, I was able to derive Given: x < or = 3 (i.e. 3,2,1....) statement 1) derives finally that x > or = 3 (i.e. 3,4,5...

by gmatwarroom

Sat Sep 29, 2012 10:53 am
Forum: Data Sufficiency
Topic: confused with this DS
Replies: 6
Views: 1518

confused with this DS

Here is DS question: If x is an integer and 4 power x < 100, what is x? (1) 4 power (x + 1) – 4 power (x – 1) > 100 (2) 4 power (x + 1) + 4 power x > 100 My answer was E) but actual answer says each one is sufficient. My approach: from given that 4 power x < 100 ==> x < or = 3 1) derives finally...

by gmatwarroom

Wed Sep 26, 2012 2:42 pm
Forum: Data Sufficiency
Topic: confused with this DS
Replies: 6
Views: 1518

mayageorge86 -- many of the stuff from your story seem familiar to me :) your weaknesses seems mine too (never get permutation combinations questions right :( ) on top of all, for me, procrastination has been the killer -- have been planning of writing gmat since last 5 years -- went long way into r...


Sorry GMATGuruNY ... still not clear for me "From the 2³ bucket we can choose no 2's, one 2, two 2's, or three 2's, for A TOTAL OF 4 OPTIONS -- ONE MORE than 2's exponent. " From above, I infer, following as options Option-1 : 2 (i.e. min one 2) Option-2 : 2*2 (i.e. two 2) Option-3 : 2*2*...

by gmatwarroom

Mon Sep 24, 2012 2:51 pm
Forum: Problem Solving
Topic: Factor question
Replies: 10
Views: 2025

thanks Anurag, Brent and GMATGuruNY. GMATGuruNY - thanks for getting to reasoning, as that's what exactly I was looking for. But, somehow, I am still not able to get step #2 Add 1 to each exponent. When I did this, I considered only 4 choices from 2 and 4 from 3 and so 4*4 = 16 and so couldnt find a...

by gmatwarroom

Mon Sep 24, 2012 1:36 pm
Forum: Problem Solving
Topic: Factor question
Replies: 10
Views: 2025

Factor question

How many factors does 36^2 (square of 36) have?
A) 2
B) 8
C) 24
D) 25
E) 26

I got it wrong but have the answer. Can someone help with answer and good explanation?

by gmatwarroom

Sun Sep 23, 2012 8:00 pm
Forum: Problem Solving
Topic: Factor question
Replies: 10
Views: 2025

need some explanation - Inequalities

Hi folks, this ques is from OG 12th edition, Q# 130. I cannot draw line here but it has a number line highlighted from -5 to +3 and ques is: which of the following is an algebraic equation of that: I chose answer B) |x| <=5 answer that OG gave, E) |x+1| <=4 my reasoning is --> x ranges from -5 to +3...

by gmatwarroom

Sat Jun 23, 2012 2:07 pm
Forum: Problem Solving
Topic: need some explanation - Inequalities
Replies: 1
Views: 804

need some explanation?

This is data sufficiency Ques:
If x and y are non zero integers, is x power y < y power x?

1) x = y power 2
2) y > 2

Appreciate some good explanation.

By the way, this is OG (2nd edition) Q 121 -- explanation there is too complex.. any simpleer approach ?

by gmatwarroom

Sun May 06, 2012 3:24 pm
Forum: Problem Solving
Topic: need some explanation?
Replies: 1
Views: 876

Hi Mitch, thanks for the formula -- helps. However, even I have the same question that Krishna asked -- pls let us know. Secondly, going by the formula, if 79 is the # of terms in the series then the greatest value (in the series) is straight to find i.e. 79*2 = 158 (as we are dealing with consecuti...

by gmatwarroom

Sun May 06, 2012 3:13 pm
Forum: Problem Solving
Topic: Something wrong with this question?
Replies: 5
Views: 1007

Something wrong with this question?

I found this question-n-answer on net and somehow not convinced with the question -- seems question was not put "accurately", not the way it was solved. ****************************************** Ques: The sum of the even numbers between 1 and n is 79*80, where n is an odd number, then n=?...

by gmatwarroom

Sat May 05, 2012 11:53 am
Forum: Problem Solving
Topic: Something wrong with this question?
Replies: 5
Views: 1007